MIE 2016/Day 1/Problem 2

Revision as of 21:22, 7 January 2018 by Anishanne (talk | contribs) (Created page with "===Problem 2=== The following system has <math>k</math> integer solutions. We can say that: <math>\begin{cases}\frac{x^2-2x-14}{x}>3\\\\x\leq12\end{cases}</math> (a) <math>...")
(diff) ← Older revision | Latest revision (diff) | Newer revision → (diff)

Problem 2

The following system has $k$ integer solutions. We can say that:

$\begin{cases}\frac{x^2-2x-14}{x}>3\\\\x\leq12\end{cases}$


(a) $0\leq k\leq 2$

(b) $2\leq k\leq 4$

(c) $4\leq k\leq6$

(d) $6\leq k\leq8$

(e) $k\geq8$


Solution 2

See Also